Difference between revisions of "1998 USAMO Problems/Problem 1"
m |
m (fixed latex) |
||
Line 4: | Line 4: | ||
== Solution == | == Solution == | ||
− | If <math>S=|a_1-b_1|+|a_2-b_2|+\cdots +|a_{999}-b_{999}|</math>, then <math>S \equiv 1+1+\cdots + 1 \equiv 999 \equiv 4 | + | If <math>S=|a_1-b_1|+|a_2-b_2|+\cdots +|a_{999}-b_{999}|</math>, then <math>S \equiv 1+1+\cdots + 1 \equiv 999 \equiv 4 \bmod{5}</math>. |
− | For integers M, N we have <math>|M-N| \equiv M-N \equiv M+N | + | For integers M, N we have <math>|M-N| \equiv M-N \equiv M+N \bmod{2}</math>. |
− | So we also have <math>S \equiv a_1+b_1+a_2-b_2+\cdots +a_{999}+b_{999} \equiv 1+2+ \cdots +1998 \equiv 999*1999 \equiv 1 | + | So we also have <math>S \equiv a_1+b_1+a_2-b_2+\cdots +a_{999}+b_{999} \equiv 1+2+ \cdots +1998 \equiv 999*1999 \equiv 1 \bmod{2}</math> also, so <math>S \equiv 9\bmod{10}</math>. |
==See Also== | ==See Also== |
Revision as of 08:02, 13 September 2012
Problem
Suppose that the set has been partitioned into disjoint pairs () so that for all , equals or . Prove that the sum ends in the digit .
Solution
If , then .
For integers M, N we have .
So we also have also, so .
See Also
1998 USAMO (Problems • Resources) | ||
Preceded by First Problem |
Followed by Problem 2 | |
1 • 2 • 3 • 4 • 5 • 6 | ||
All USAMO Problems and Solutions |